Saltar al contenido principal
LibreTexts Español

7.5.E: Problemas en las Premedidas y Temas Relacionados

  • Page ID
    114003
  • \( \newcommand{\vecs}[1]{\overset { \scriptstyle \rightharpoonup} {\mathbf{#1}} } \) \( \newcommand{\vecd}[1]{\overset{-\!-\!\rightharpoonup}{\vphantom{a}\smash {#1}}} \)\(\newcommand{\id}{\mathrm{id}}\) \( \newcommand{\Span}{\mathrm{span}}\) \( \newcommand{\kernel}{\mathrm{null}\,}\) \( \newcommand{\range}{\mathrm{range}\,}\) \( \newcommand{\RealPart}{\mathrm{Re}}\) \( \newcommand{\ImaginaryPart}{\mathrm{Im}}\) \( \newcommand{\Argument}{\mathrm{Arg}}\) \( \newcommand{\norm}[1]{\| #1 \|}\) \( \newcommand{\inner}[2]{\langle #1, #2 \rangle}\) \( \newcommand{\Span}{\mathrm{span}}\) \(\newcommand{\id}{\mathrm{id}}\) \( \newcommand{\Span}{\mathrm{span}}\) \( \newcommand{\kernel}{\mathrm{null}\,}\) \( \newcommand{\range}{\mathrm{range}\,}\) \( \newcommand{\RealPart}{\mathrm{Re}}\) \( \newcommand{\ImaginaryPart}{\mathrm{Im}}\) \( \newcommand{\Argument}{\mathrm{Arg}}\) \( \newcommand{\norm}[1]{\| #1 \|}\) \( \newcommand{\inner}[2]{\langle #1, #2 \rangle}\) \( \newcommand{\Span}{\mathrm{span}}\)\(\newcommand{\AA}{\unicode[.8,0]{x212B}}\)

    Ejercicio\(\PageIndex{1}\)

    Rellene los datos que faltan en las pruebas, notas y ejemplos de esta sección.

    Ejercicio\(\PageIndex{2}\)

    Describir\(m^{*}\) el\(2^{S}\) inducido por una premedida\(\mu : \mathcal{C} \rightarrow E^{*}\) tal que cada uno de los siguientes sostenga.
    (a)\(\mathcal{C}=\{S, \emptyset\}, \mu S=1\).
    b)\(\mathcal{C}=\{S, \emptyset, \text {and all singletons}\}; \mu S=\infty, \mu\{x\}=1\).
    (c)\(\mathcal{C}\) como en (b), con\(S\) incontables;\(\mu S=1,\) y de\(\mu X=0\) otra manera.
    d)\(\mathcal{C}=\{\text {all proper subsets of } S\}; \mu X=1\) cuando\(\emptyset \subset X \subset S; \mu \emptyset=0\).

    Ejercicio\(\PageIndex{3}\)

    Demostrar que las premedidas
    \[v^{\prime} : \mathcal{C}^{\prime} \rightarrow[0, \infty]\]
    inducen una y la misma medida externa (Lebesgue)\(m^{*}\) en\(E^{n},\) con\(v^{\prime}=v\) (volumen, como en §2):
    (a)\(\mathcal{C}^{\prime}=\{\text {open intervals}\}\);
    (b); (c)\(\mathcal{C}^{\prime}=\{\text {half-open intervals}\}\);
    (d)\(\mathcal{C}^{\prime}=\{\text {closed intervals}\}\);
    (d)\(\mathcal{C}^{\prime}=\mathcal{C}_{\sigma}\);
    e)\(\mathcal{C}^{\prime}=\{\text {open sets}\}\); f
    )\(\mathcal{C}^{\prime}=\{\text {half-open cubes}\}\);
    [Insinúa: a)\(m^{\prime}\) Sea la medida externa\(v^{\prime}\) -inducida; let\(\mathcal{C}=\{\text {all intervals}\}.\) As\(\mathcal{C}^{\prime} \subseteq \mathcal{C}, m^{\prime} A \geq m^{*} A.\) (¿Por qué?) Además,
    \[(\forall \varepsilon>0)\left(\exists\left\{B_{k}\right\} \subseteq \mathcal{C}\right) \quad A \subseteq \bigcup_{k} B_{k} \text { and } \sum v B_{k} \leq m^{*} A+\varepsilon.\]
    (¿Por qué?) Por Lema 1 en §2,
    \[\left(\exists\left\{C_{k}\right\} \subseteq \mathcal{C}^{\prime}\right) \quad B_{k} \subseteq C_{k} \text { and } v B_{k}+\frac{\varepsilon}{2^{k}}>v^{\prime} C_{k}.\]
    Deducir eso\(m^{*} A \geq m^{\prime} A, m^{*}=m^{\prime}\). De igual manera para (b) y (c). Para (d), use Corolario 1 y Nota 3 en §1. Para (e), use Lemma 2 en §2. Para (f), use Problema 2 en §2.]

    Ejercicio\(\PageIndex{3'}\)

    Hacer Problema 3 (a) - (c), con\(m^{*}\) sustituido por el contenido exterior de Jordania\(c^{*}\) (Nota 6).

    Ejercicio\(\PageIndex{4}\)

    Hacer el Problema 3, con\(v\) y\(m^{*}\) reemplazado por la premedida LS y la medida exterior. (Use el Problema 7 en §4.)

    Ejercicio\(\PageIndex{5}\)

    Demuestre que un conjunto\(A \subseteq E^{n}\) está acotado si su contenido exterior de Jordania es finito.

    Ejercicio\(\PageIndex{6}\)

    Encuentra un conjunto\(A \subseteq E^{1}\) tal que
    (i) su medida exterior de Lebesgue sea\(0\)\(\left(m^{*} A=0\right),\) mientras que su contenido exterior de Jordania\(c^{*} A=\infty\);
    (ii)\(m^{*} A=0, c^{*} A=1\) (ver Corolario 6 en §2).

    Ejercicio\(\PageIndex{7}\)


    \[\mu_{1}, \mu_{2} : \mathcal{C} \rightarrow[0, \infty]\]
    Dejen entrar dos premedidas\(S\) y dejar\(m_{1}^{*}\) y\(m_{2}^{*}\) ser las medidas exteriores inducidas por ellas.
    Demuestre que si está\(m_{1}^{*}=m_{2}^{*}\)\(m_{1}^{*}=m_{2}^{*}\) encendido\(\mathcal{C},\) entonces en todos\(2^{S}\).

    Ejercicio\(\PageIndex{8}\)

    Con la notación de Definición 3 y Nota 6, probar lo siguiente.
    (i) Si\(A \subseteq B \subseteq S\) y\(m^{*} B=0,\) luego de\(m^{*} A=0;\) manera similar para\(c^{*}\).
    [Pista: Usar monotonicidad.]
    (ii) La familia establecida
    \[\left\{X \subseteq S | c^{*} A=0\right\}\]
    es un anillo conjunto hereditario, es decir, un anillo\(\mathcal{R}\) tal que
    \[(\forall B \in \mathcal{R})(\forall A \subseteq B) \quad A \in \mathcal{R}.\]
    (iii) La familia establecida
    \[\left\{X \subseteq S | m^{*} X=0\right\}\]
    es un\(\sigma\) anillo hereditario.
    (iv) Así también\(\mathcal{H}\) es
    \[\mathcal{H}=\{\text {those } X \subseteq S \text { that have basic coverings}\};\]
    así el\(\sigma\) anillo hereditario generado por\(\mathcal{C}\) (ver Problema 14 en §3).

    Ejercicio\(\PageIndex{9}\)

    Continuando Problema 8 (iv), probar que si\(\mu\) es\(\sigma\) -finito (Definición 4), así es\(m^{*}\) cuando se restringe a\(\mathcal{H}.\)
    Mostrar, además, que si\(\mathcal{C}\) es un semiring, entonces cada uno\(X \in \mathcal{H}\) tiene una cobertura básica\(\left\{Y_{n}\right\},\) con\(m^{*} Y_{n}<\infty\) y con todos\(Y_{n}\) disjuntos.
    [Pista: Mostrar que
    \[X \subseteq \bigcup_{n=1}^{\infty} \bigcup_{k=1}^{\infty} B_{n k}\]
    para algunos conjuntos\(B_{n k} \in \mathcal{C},\) con\(\mu B_{n k}<\infty.\) Luego use la Nota 4 en §5 y el Corolario 1 de §1.]

    Ejercicio\(\PageIndex{10}\)

    Mostrar que si
    \[s : \mathcal{C} \rightarrow E^{*}\]
    es\(\sigma\) -finito y aditivo en\(\mathcal{C},\) un semiring, entonces el\(\sigma\) -anillo\(\mathcal{R}\) generado por\(\mathcal{C}\) es igual al\(\sigma\) -anillo\(\mathcal{R}^{\prime}\) generado por
    \[\mathcal{C}^{\prime}=\{X \in \mathcal{C}| | s X |<\infty\}\]
    (cf. Problema 6 en §4).
    [Pista: Por\(\sigma\) -finitud,
    \[(\forall X \in \mathcal{C})\left(\exists\left\{A_{n}\right\} \subseteq \mathcal{C}| | s A_{n} |<\infty\right) \quad X \subseteq \bigcup_{n} A_{n};\]
    entonces
    \[X=\bigcup_{n}\left(X \cap A_{n}\right), \quad X \cap A_{n} \in \mathcal{C}^{\prime}.\]
    (Use Lema 3 en §4.)
    Así\((\forall X \in \mathcal{C}) X\) es una unión contable de\(\mathcal{C}^{\prime}\) -conjuntos; así\(\mathcal{C} \subseteq \mathcal{R}^{\prime}.\) Deduce\(\mathcal{R} \subseteq \mathcal{R}^{\prime}\). Proceder.]

    Ejercicio\(\PageIndex{11}\)

    Con todo como en el Teorema 3, demostrar que si\(A\) tiene coberturas básicas, entonces
    \[\left(\exists B \in \mathcal{A}_{\delta}\right) \quad A \subseteq B \text { and } m^{*} A=m^{*} B.\]
    [Pista: Por fórmula (4),
    \[(\forall n \in N)\left(\exists X_{n} \in \mathcal{A} | A \subseteq X_{n}\right) \quad m^{*} A \leq m X_{n} \leq m^{*} A+\frac{1}{n}.\]
    (¡Explique!) Establecer
    \[B=\bigcap_{n=1}^{\infty} X_{n} \in \mathcal{A}_{\delta}.\]
    Proceder. Para\(\mathcal{A}_{\delta},\) ver Definición 2 (b) en §3.]

    Ejercicio\(\PageIndex{12}\)

    Dejar\((S, \mathcal{C}, \mu)\) y\(m^{*}\) ser como en la Definición 3. Mostrar que si\(\mathcal{C}\) es un\(\sigma\) campo -en\(S,\) entonces
    \[(\forall A \subseteq S)(\exists B \in \mathcal{C}) \quad A \subseteq B \text { and } m^{*} A=\mu B.\]
    [Pista: Use Problema 11 y Nota 3.]

    Ejercicio\(\PageIndex{13}\)

    \(\Rightarrow^{*}\)Mostrar que si
    \[s : \mathcal{C} \rightarrow E\]
    es\(\sigma\) -finito y\(\sigma\) -aditivo en\(\mathcal{C},\) un semiring, entonces\(s\) tiene como máximo una extensión\(\sigma\) -aditiva al\(\sigma\) anillo\(\mathcal{R}\) -generado por\(\mathcal{C}.\)
    (Tenga en cuenta que\(s\) es automáticamente\(\sigma\) -finito si es finito, por ejemplo, complejo o valor vectorial.)
    [Esquema: Let
    \[s^{\prime}, s^{\prime \prime} : \mathcal{R} \rightarrow E\]
    be two\(\sigma\) -aditive extensions of\(s.\) By Problem 10, también\(\mathcal{R}\) es generado por
    \[\mathcal{C}^{\prime}=\{X \in \mathcal{C}| | s X |<\infty\}.\]
    Now set
    \[\mathcal{R}^{*}=\left\{X \in \mathcal{R} | s^{\prime} X=s^{\prime \prime} X\right\}.\]
    Mostrar que\(\mathcal{R}^{*}\) satisface propiedades (i) - (iii) del Teorema 3 en §3, con\(\mathcal{C}\) sustituidas por\(\mathcal{C}^{\prime};\) so\(\mathcal{R}=\mathcal{R}^{*}\).]

    Ejercicio\(\PageIndex{14}\)

    Let\(m_{n}^{*}(n=1,2, \ldots)\) Ser medidas exteriores en\(S\) tal que
    \[(\forall X \subseteq S)(\forall n) \quad m_{n}^{*} X \leq m_{n+1}^{*} X.\]
    Set
    \[\mu^{*}=\lim _{n \rightarrow \infty} m_{n}^{*}.\]
    Show que\(\mu^{*}\) es una medida exterior en\(S\) (ver Nota 5).

    Ejercicio\(\PageIndex{15}\)

    \((S, \rho)\)Se dice que una medida exterior\(m^{*}\) en un espacio métrico tiene la propiedad Carathéodory (CP) iff
    \[m^{*}(X \cup Y) \geq m^{*} X+m^{*} Y\]
    siempre que\(\rho(X, Y)>0,\) donde
    \[\rho(X, Y)=\inf \{\rho(x, y) | x \in X, y \in Y\}.\]
    Para tal\(m^{*},\) probar que
    \[m^{*}\left(\bigcup_{k} X_{k}\right)=\sum_{k} m^{*} X_{k}\]
    si\(\left\{X_{k}\right\} \subseteq 2^{S}\) y
    \[\rho\left(X_{i}, X_{k}\right)>0 \quad(i \neq k).\]
    [Pista: Para uniones finitas, use el CP, la subaditividad y la inducción. Deducir que
    \[(\forall n) \sum_{k=1}^{n} m^{*} X_{k} \leq m^{*} \bigcup_{k=1}^{\infty} X_{k}.\]
    Dejemos\(n \rightarrow \infty.\) Proceder.]

    Ejercicio\(\PageIndex{16}\)

    Dejar\((S, \mathcal{C}, \mu)\) y\(m^{*}\) ser como en la Definición 3, con\(\rho\) una métrica para\(S.\) Let\(\mu_{n}\) be la restricción de\(\mu\) a la familia\(\mathcal{C}_{n}\)\(X \in \mathcal{C}\) de todos los diámetros
    \[d X \leq \frac{1}{n}.\]
    Dejar\(m_{n}^{*}\) ser la medida exterior\(\mu_{n}\) -inducida en \(S.\)
    Demostrar que
    (i)\(\left\{m_{n}^{*}\right\} \uparrow\) como en el Problema 14;
    (ii) la medida externa
    \[\mu^{*}=\lim _{n \rightarrow \infty} m_{n}^{*}\]
    tiene el CP (ver Problema 15), y
    \[\mu^{*} \geq m^{*} \text { on } 2^{S}.\]
    [Esquema: Vamos\(\rho(X, Y)>\varepsilon>0(X, Y \subseteq S)\).
    Si para algunos no\(n, X \cup Y\) tiene cobertura básica a partir de\(\mathcal{C}_{n},\) entonces
    \[\mu^{*}(X \cup Y) \geq m_{n}^{*}(X \cup Y)=\infty \geq \mu^{*} X+\mu^{*} Y,\]
    y el CP sigue. (¡Explique!)
    Así suponemos
    \[\left(\forall n>\frac{1}{\varepsilon}\right)(\forall k)\left(\exists B_{n k} \in \mathcal{C}_{n}\right) \quad X \cup Y \subseteq \bigcup_{k=1}^{\infty} B_{n k}.\]
    Uno puede elegir el\(B_{n k}\) para que
    \[\sum_{k=1}^{\infty} \mu B_{n k} \leq m_{n}^{*}(X \cup Y)+\varepsilon.\]
    (¿Por qué?) Como
    \[d B_{n k} \leq \frac{1}{n}<\varepsilon,\]
    algunos\(B_{n k}\) cubren\(X\) solamente, otros\(Y\) solamente. (¿Por qué?) Deducir que
    \[\left(\forall n>\frac{1}{\varepsilon}\right) \quad m_{n}^{*} X+m_{n}^{*} Y \leq \sum_{k=1}^{\infty} \mu_{n} B_{n k} \leq m_{n}^{*}(X \cup Y)+\varepsilon.\]
    Vamos\(\varepsilon \rightarrow 0\) y luego\(n \rightarrow \infty\).
    Además,\(m^{*} \leq m_{n}^{*} \leq \mu^{*}.\) (¿Por qué?)]

    Ejercicio\(\PageIndex{17}\)

    Continuando Problema 16, supongamos que
    \((\forall \varepsilon>0)(\forall n, k)(\forall B \in \mathcal{C})\left(\exists B_{n k} \in \mathcal{C}_{n}\right)\)
    \[B \subseteq \bigcup_{k=1}^{\infty} B_{n k} \text { and } \mu B+\varepsilon \geq \sum_{k=1}^{\infty} \mu B_{n k}.\]
    Mostrar que
    \[m^{*}=\lim _{n \rightarrow \infty} \mu_{n}^{*}=\mu^{*},\]
    así\(m^{*}\) mismo tiene el CP.
    [Consejos: basta con probar que\(m^{*} A \geq \mu^{*} A\) si\(m^{*} A<\infty.\) (¿Por qué?)
    Ahora, dado\(\varepsilon>0, A\) tiene una cobertura
    \[\left\{B_{i}\right\} \subseteq c\]
    tal que
    \[m^{*} A+\varepsilon \geq \sum \mu B_{i}.\]
    (¿Por qué?) Por suposición,
    \[(\forall n) \quad B_{i} \subseteq \bigcup_{k=1}^{\infty} B_{n k}^{i} \in \mathcal{C}_{n} \text { and } \mu B_{i}+\frac{\varepsilon}{2^{i}} \geq \sum_{k=1}^{\infty} \mu B_{n k}^{i}.\]
    Deducir que
    \[m^{*} A+\varepsilon>\sum \mu B_{i} \geq \sum_{i=1}^{\infty}\left(\sum_{k=1}^{\infty} \mu B_{n k}^{i}-\frac{\varepsilon}{2^{i}}\right)=\sum_{i, k} \mu B_{n k}^{i}-\varepsilon \geq m_{n}^{*} A-\varepsilon.\]
    Dejemos\(\varepsilon \rightarrow 0;\) entonces\(n \rightarrow \infty\).]

    Ejercicio\(\PageIndex{18}\)

    Usando el Problema 17, muestran que las medidas exteriores de Lebesgue y Lebesgue-Stieltjes tienen el CP.


    7.5.E: Problemas en las Premedidas y Temas Relacionados is shared under a CC BY 1.0 license and was authored, remixed, and/or curated by LibreTexts.